Need help with this

Find each indicated measure for circle O

Answers

Answer 1

Answer:

Angle A = 41, Arc CE = 42, Angle C = 41, Angle D = 40, Angle ABE = 61

Step-by-step explanation:

Angle A = Arc BD/2  (Inscribed Angle)

Angle A = 82/2

Angle A = 41

Arc CE = 2 x Angle CEB (Inscribed Angle)

Arc CE = 2 x 21

Arc CE = 42

Angle C = Arc BD/2 (Inscribed Angle)

Angle C = 82/2

Angle C = 41

Angle D = Arc AC/2  (Inscribed Angle)

Angle D = 80/2

Angle D = 40

Angle ABE = Arc AE/2

Angle ABE = Arc AC + Arc CE/2

Angle ABE = 122/2

Angle ABE = 61

Answer 2

m < A =  [tex]39^{0}[/tex] , mCE =  [tex]40^{0}[/tex], m < c = [tex]39^{0}[/tex], m < D = [tex]37^{0}[/tex] m < ABE = [tex]57^{0}[/tex]

What are angles in a circle ?

A circle has a total of 360 degrees all the way around the center, so if that central angle determining a sector has an angle measure of 60 degrees, then the sector takes up 60/360 or 1/6, of the degrees all the way around.

a. m < A = [tex]\frac{1}{2}[/tex] mBD

= [tex]\frac{1}{2}[/tex] × [tex]78^{0}[/tex]

= [tex]39^{0}[/tex]

b. mCE = 2m < CBE

= 2 × [tex]20^{0}[/tex]

= [tex]40^{0}[/tex]

c. m < c = m < A = [tex]39^{0}[/tex]

d. m < D = [tex]\frac{1}{2}[/tex] mAC

= [tex]\frac{1}{2}[/tex] × [tex]74^{0}[/tex]

= [tex]37^{0}[/tex]

e. m < ABE = [tex]\frac{1}{2}[/tex] mAC + [tex]\frac{1}{2}[/tex] mCE

= [tex]37^{0}[/tex] + [tex]20^{0}[/tex]

= [tex]57^{0}[/tex]

Hence, m < A =  [tex]39^{0}[/tex] , mCE =  [tex]40^{0}[/tex], m < c = [tex]39^{0}[/tex], m < D = [tex]37^{0}[/tex] m < ABE = [tex]57^{0}[/tex]

Learn more about angles in a circle here

https://brainly.com/question/23247585

#SPJ2


Related Questions

A ball is thrown up in the air, and it's height (in feet) as a function of time (in seconds) can be written as h(t) = -16t2 + 32t + 6. Describe in detail (using formulas and/or specific steps needed) TWO ways to find the maximum height reached by the ball.

Answers

Answer:

1) Using properties of the quadratic equation:

Here the height equation is:

h(t) = -16t^2 + 32t + 6

We can see that the leading coefficient is negative, this means that the arms of the graph will open downwards.

Then, the vertex of the quadratic equation will be the maximum.

Remember that for a general quadratic equation:

a*x^2 + b*x + c = y

the x-value of the vertex is:

x = -b/2a

Then in our case, the vertex is at:

t = -32/(2*-16) = 1

The maximum height will be the height equation evaluated in this time:

h(1) = -16*1^2 + 32*1 + 6 = 22

The maximum height is 22ft

2) Second method, using physics.

We know that an object reaches its maximum height when the velocity is equal to zero (the velocity equal to zero means that, at this point, the object stops going upwards).

If the height equation is:

h(t) = -16*t^2 + 32*t + 6

the velocity equation is the first derivation of h(t)

Remember that for a function f(x) = a*x^n

we have that:

df(x)/dx = n*a*x^(n-1)

Then:

v(t) = dh(t)/dt = 2*(-16)*t + 32 + 0

v(t) = -32*t + 32

Now we need to find the value of t such that the velocity is equal to zero:

v(t) = 0 = -32*t + 32

       32*t = 32

            t = 32/32 = 1

So the maximum height is at t = 1

(same as before)

Now we just need to evaluate the height equation in t = 1:

h(1) = -16*1^2 + 32*1 + 6 = 22

The maximum height is 22ft

Help please me please please need explanation

Answers

9514 1404 393

Answer:

  a) see attached

  b) 1/3

Step-by-step explanation:

Your grid doesn't come with any coordinate axes, so you will need to draw those and label them. You will also need to determine the number of grid squares for each unit. (I suggest 2 grid squares per unit to give you a little more distance between the points you plot. The attachment shows 5 small squares per unit. You may not have this much space on your graph.)

Plot the points. Once you do this, you can see that their horizontal difference is 3 units and their vertical difference is 1 unit. The right point is above the left point, so the slope is positive:

  m = (rise)/(run) = 1/3 . . . slope of the line

Math question please show work due today last day to submit Assignments or I fail please help

Answers

Answer:

The polygons are similar.

Step-by-step explanation:

By definition, the corresponding sides and angles of similar polygons must be in some constant proportion.

Therefore, we can find the ratio between corresponding sides to see if that ratio is maintained.

The side marked 3 cm in the smaller figure corresponds with the side marked 3.75 cm in the larger figure. This proportion is [tex]\frac{3}{3.75}[/tex].

The side marked 5 in the smaller figure corresponds with the side marked 6.25 cm in the larger figure. This proportion is [tex]\frac{5}{6.25}[/tex].

If these polygons are similar, these proportions must be equal.

[tex]\frac{3}{3.75}=\frac{5}{6.25},\\0.8=0.8\:\checkmark[/tex]

Checking, we see that these proportions are equal, and therefore the polygons are similar.

James is using cement to make a new sidewalk and new steps.
Part A
He will make a new rectangular sidewalk that is 8 feet long, 4 feet wide, and 0.25 foot thick.
What is the volume, in cubic feet, of the cement used to make the sidewalk? Show or explain all of the steps you used to determine your answer.

Part B
This diagram shows James’s design for the steps. The height of each step will be 6 inches. All angles in the diagram are right angles.
I need it now please!

Answers

The volume, of the cement used to make the sidewalk is 8 ft³

What is volume?

Volume is defined as the space occupied within the boundaries of an object in three-dimensional space. It is also known as the capacity of the object.

Given that, James is using cement to make a new sidewalk and new steps,

He will make a new rectangular sidewalk that is 8 feet long, 4 feet wide, and 0.25 foot thick.

Volume = length x width x height

Volume = 8 x 4 x 0.25 = 8 ft³

Hence, the volume, of the cement used to make the sidewalk is 8 ft³

Learn more about volumes, click;

https://brainly.com/question/1578538

#SPJ1

Match each expression with its sum or difference

Answers

Answer:

1-4=-3

-1-4=-5

-1+4=3

1-|-4|=5

Answer:

[tex]1-4=-3\\-1-4=-5\\-1+4=3\\1-(-4)=5[/tex]

Step-by-step explanation:

The hardest part about negative numbers in addition is remembering when to add, rather than subtract. In the last equation, we add the numbers together, because the negative sign cancels out with subtraction. All of the other equations we can follow exactly what it says to do, because there are no minus signs right next to each other.

Tao uses elimination to solve the following system of linear equations.

Answers

Given:

The system of equation is:

[tex]-2x+4y=16[/tex]

[tex]2x+2y=8[/tex]

To find:

The solution of the given system of equations.

Solution:

We have,

[tex]-2x+4y=16[/tex]              ...(i)

[tex]2x+2y=8[/tex]                ...(ii)

Add (i) and (ii) to eliminate the variable x.

[tex]6y=24[/tex]

[tex]y=\dfrac{24}{6}[/tex]

[tex]y=4[/tex]

Putting [tex]y=4[/tex] in (ii), we get

[tex]2x+2(4)=8[/tex]

[tex]2x+8=8[/tex]

[tex]2x=8-8[/tex]

[tex]2x=0[/tex]

Divide both sides by 2.

[tex]x=0[/tex]

Therefore, the solution of given system of equations is (0,4). Hence, the correct option is A.

Answer:

(0,4) or A

Step-by-step explanation:

Edge 2021

What is the value of the product (3 - 2i)(3 + 2i)

Answers

Answer:

13

Step-by-step explanation:

(3 - 2i)(3 + 2i)

FOIL

first  3*3 = 9

outer 3*2i = 6i

inner -2i*3 = -6i

last  -2i * 2i = -4i^2 = -4(-1) =4

Add together

9+6i-6i+4 = 13

30 pts 5 stars thanks and brainliest please- this question is really hard...

Answers

Answer:

From least to greatest

C

B

A

Step-by-step explanation:

Function A is the only one with a positive y-intercept

so it's the greatest

Function B has a y-intercept of -1 as read of the graph

it is the middle one

Function C has a y-intercept of -9/4 as given in point (0, -9/4)

this is the least

-----------------

From least to greatest

C

B

A

−n+(−4)−(−4n)+6 find equivalent expressions PLZ

Answers

Answer:

3n+2

Step-by-step explanation:

−n+(−4)−(−4n)+6

Combine like terms

−n−(−4n)+6+(−4)

A negative negative is like adding

−n+(4n)+6+(-4)

3n+2

The answer to your question is 3n+2

3
33%
A fair dice has six sides, numbered 1 to 6
After it is rolled, five of the numbers can be seen.
ws
a) Write down the probability that one of these five numbers is 3
(1)
b) Work out the least possible sum of the five numbers.
(2)

Answers

Answer:

a) 5/6 probability that one of the five numbers is 3

b) the least possible sum of the five numbers will be 15

as the least will come from the lowest 5 numbers

you have at most $20.25 to spend at a store. Shirts cost $11.50 each, and pants cost $16 each. Let s be numbers of shirts and p be number of pants. Write an inequality that represents the numbers of shirts and pants you can afford. DO NOT INCLUDE THE DOLLAR SIGN IN YOU INEQUALITY. An inequality is______

Answers

is this an algebra class? if so it just

11.50x + 16y = 20.25

Solve for. . Write your answer as “x=__”

Answers

Answer:

x=50

Step-by-step explanation:

Since it's a straight line, 3x-20+x=180. 4x=200, x=50

A particle is moving with acceleration a ( t ) = 18 t + 16 . Its position at time t = 0 is s ( 0 ) = 10 and its velocity at time t = 0 is v ( 0 ) = 16 . What is its position at time t = 8 ?

Answers

Answer:

[tex]s(8)=2186[/tex]

Step-by-step explanation:

A particle is moving with acceleration modeled by the function:

[tex]a(t)=18t+16[/tex]

We are given that its position s(t) at t = 0 is 10 and that its velocity v(t) at t = 0 is 16.

And we want to find its position at t = 8.

Velocity is the integral of the acceleration. Hence:

[tex]\displaystyle v(t)=\int a(t)\, dt=\int 18t+16\, dt[/tex]

Find the velocity. Remember the constant of integration!

[tex]v(t)=9t^2+16t+C[/tex]

Since v(t) is 16 when t = 0:

[tex](16)=9(0)^2+16(0)+C\Rightarrow C=16[/tex]

Hence, our velocity is given by:

[tex]v(t)=9t^2+16t+16[/tex]

Position is the integral of the velocity. Hence:

[tex]\displaystyle s(t)=\int v(t)\, dt=\int 9t^2+16t+16\, dt[/tex]

Integrate:

[tex]\displaystyle s(t)=3t^3+8t^2+16t+C[/tex]

s(t) is 10 when t = 10. Hence:

[tex]C=10[/tex]

So, our position function is:

[tex]s(t)=3t^3+8t^2+16t+10[/tex]

The position at t = 8 will be:

[tex]s(8)=3(8)^3+8(8)^2+16(8)+10=2186[/tex]

any method that uses formulas, figures, or proportions to measure an object Is a(n) ______?

Answers

Answer:

Indirect measurement

Step-by-step explanation:

Maria's house is due south of Boise and due east of marching. Mamas house is 12 miles from Boise and 20 miles are marching. How far apart are Boise in marching as the crow flies? Round your answer to the nearest 10th

Answers

Answer:

23.32 Miles

Step-by-step explanation:

According to the Question,

Given, Maria's house is due south of Boise and due east of marching & Maria's house is 12 miles from Boise and 20 miles From marching .

(For Diagram Please Find in Attachment)

In Triangle ABC , Apply Pythagoras Theorem, We get

AC²=AB² + BC²

AC² = 12² + 20²

AC² = 544 ⇒ √544 ⇔ 23.32 miles

??!?!?!?!?!!?!?!?!!?!!!?!!!!?

Answers

Answer:

The third option, a rational number

like 1/5 + 2.5 is still rational (but neither irrational nor whole/an integer)

it would be 0.2 + 2.5 = 2.7 btw

or

1/5 + 5/2

= 2/10 + 25/10

= 27/10

Uadrilateral ABCD is dilated at center (0,0) with scale factor 1/2 to form quadrilateral A'B'C'D'. What is the length of A'B'?

Answers

Answer:

i think isA

Step-by-step explanation:


Need help finding the cost! ASAP!

Answers

Answer:

Apple Cost: $1.10

Orange Cost: $1.05

Three oranges and two apples cost: $5.35

Step-by-step explanation:

answer:
first one: 1.10$
second one: 1.05$
total: 5.35$

What number makes the expression a perfect square? X squared -12x +

Answers

Answer:

36

Step-by-step explanation:

x²-12x+36 = (x-6)(x-6)

2 of 8
Given that y = 8 cm and 0 = 44", work out a rounded to 1 DP.
Oo
The diagram is not drawn accurately.
2=
cm
PLEASE HELP

Answers

Answer:

x ≈ 5.6 cm

Step-by-step explanation:

Using the sine ratio in the right triangle

sin44° = [tex]\frac{opposite}{hypotenuse}[/tex] = [tex]\frac{x}{y}[/tex] = [tex]\frac{x}{8}[/tex] ( multiply both sides by 8 )

8 × sin44° = x , then

x ≈ 5.6 cm ( to 1 dec. place )

IMA In a certain Algebra 2 class of 23 students, 7 of them play basketball and 12 of them play baseball. There are 9 students who play neither sport. What is the probability that a student chosen randomly from the class plays basketball or baseball​

Answers

Answer:

19/28 or about 68%

Step-by-step explanation:

7 + 12 + 9 = 28

7 + 12/28

19/28

Which choice is equivalent to the product below when x is greater than 0?

Answers

I think that's ur answer

Help ASAP !!!

What is the angle of 1

Answers

Answer:

100°

Step-by-step explanation:

m L 1 = (180-100) + 20 = 80+20 = 100°

Find m∠1 and m∠2. Tell which theorem can be used.

Answers

Answer:

1 = 117 ( vertically opposite angles are equal )

2 = 117 ( corresponding angles are always equal )

Step-by-step explanation:

Answer:

angle one=122 because it is using the alternate interior angle theorem

angle 2=58 because it is using the consecutive interior angle theorem

hope this helps

have a good day :)

Step-by-step explanation:

remember consecutive=same sine 122 is on the same side of angle 2 just substract 180-122=58

since 122 is on the opposite side of angle 1 it's going to be alternate something so since it's on the inside its going to be alternate interior angle

-4.3 + x equals 0 selected place on the number line to positive value of x​

Answers

Answer:

x is equal to 4.3

Step-by-step explanation:

since it's -4.3 + x is equal to 0

you take -4.3 to the other side of the equal sign which makes it positive and x is equal to 4.3

Kendra spent ⅓ of her allowance on a book and ⅖ on a snack. If she had four dollars remaining after purchasing a book and snack, what was the total amount of her allowance?

Answers

Answer: 15$

Step-by-step explanation:

Given

Kendra spent one-third of allowance on a book and two-fifth on snack.

If she had four dollar remaining after purchase

Suppose x is the original amount

[tex]\therefore x-\dfrac{x}{3}-\dfrac{2x}{5}=4\\\\\Rightarrow \dfrac{15x-5x-6x}{15}=4\\\\\Rightarrow 4x=60\\\Rightarrow x=15[/tex]

Thus, 15$ was her total allowance.

857,963 five has a value of​

Answers

Answer: 5 has a value of 50 000 2.

4,289,815

Hope this helps ;)

18/27=1/3 9/15<4/4 3/4<2/3 5/6>11/12 which is a true statement?

Answers

Answer:

The correct answer us 9/15<4/4

Can I please have a brainliest? I want to rank up :)

I NEED HELP WITH THIS ASAP PLEASEEEE :(

Answers

Answer:

[tex]\sqrt{1}[/tex]

Step-by-step explanation:

Anything to the zeroth power becomes 1, and anything to the negative power can be turned into the root of it. For example, 1 to the -2 power = [tex]\sqrt{1\\}[/tex]

Hope this helps!!

Please mark me Brainliest

:)

What they said I believe

Find the value of x:
77.5% of x is 49

Answers

77.5/100=49/x

77.5x= 4900

x= 4900/77.5

x= 63.22 (two decimal places)

mark me brainliest pls :)))

Other Questions
Need help bad please help me Are is the Triangle SWT and triangle SVU similar? What would be the answer? Need help with this thanks Is 0.669429565254154... a rational number What does the word quietly mean?A. Quiet again B. Always quiet C. Not very quietD. In a quiet way si el radio de un circulo es de 3 pulgadas. cual es el area del circulo? Let U = {1, 2, 3, 4, 5, 6, 7), A = {2, 4, 6, 7), and B = {1, 4, 7}. Find the set (A U B)'. I need help solving this , help ! If you do this for me I will mark you as brainless Which of the following best describes the role of meiosis?1Creating egg and sperm cells (gamete sex cells).2Producing more cells for an organism to grow bigger.3Producing more organelles.4Creating new cells that are Explain why a smart meter monitors the amount of electrical energy being used as well as the time at which it is being used. The end product(s) of the light-independent reactions of photosynthesis is/are - Select all that apply.1.Glucose2.ATP3.Oxygen4.Carbon dioxide PLEASEEEEEE I REALLY NEED HELPPPP WITH THIS PLEASEEEEEE IM BEGGING SOMONEEEE PLEASEEEE PLEASEEEE PLEASEEEEEE How long did it take to rebuild the damage in Manila, Philippines caused by World War 2? (Please help I need this ansswer fast!) Solve for y:76y = 19Y =2/3Y = 1/4 merci de m'aider je suis tibo inshape et n'oubliez pas que vous tes les meilleurs boomune expression se faire une place de quoile printempsdes cordesla dernire pluieau soleil What was leopold II's contribution to the colonization of africa [Pre-Calc] Please Help! I dont know where to start. How do I do this? What is N + 9 = 27?I am confused What are the solutions of the equation 2x6=2x6 ?Select each correct answer.x=1x = 0x = 1x = 2x = 3